Official Guide Explanation:
Data Sufficiency #86

 

 

Background

This is just one of hundreds of free explanations I've created to the quantitative questions in The Official Guide for GMAT Quantitative Review (2nd ed.). Click the links on the question number, difficulty level, and categories to find explanations for other problems.

These are the same explanations that are featured in my "Guides to the Official Guide" PDF booklets. However, because of the limitations of HTML and cross-browser compatibility, some mathematical concepts, such as fractions and roots, do not display as clearly online.

Click here for an example of the PDF booklets. Click here to purchase a PDF copy.

 

Solution and Metadata

Question: 86
Page: 159
Difficulty: 5 (Moderate)
Category 1: Arithmetic > Properties of Integers > Other
Category 2: Arithmetic > Properties of Integers > Evens and Odds

Explanation: If m and n are consecutive positive integers, there are two possible scenarios: m is greater than n (as in m = 3 and n = 2) and n is greater than m (as in m = 2 and n = 3).

Statement (1) is sufficient: if m - 1 and n + 1 are consecutive, we can check the two scenarios above to see if both of them work. If m = 3 and n = 2, m - 1 = 2 and n + 1 = 3, which are consecutive integers, so it is possible that m is greater than n. If m = 2 and n = 3, m - 1 = 1 and n + 1 = 4, which are not consecutive integers. Thus, m must be greater than n.

Statement (2) is insufficient: if m = 2, n could be either 1 or 3: greater than or less than m. Choice (A) is correct.

Click here for the full list of GMAT Quant Review explanations.

 

You should follow me on Twitter. While you're at it, take a moment to subscribe to GMAT Hacks via RSS or Email.

Total GMAT Math

The comprehensive guide to the GMAT Quant section. It's "far and away the best study material available," including over 300 realistic practice questions and more than 500 exercises!
Click to read more.